PLEASE ANSWER ASAP
1. How many atoms are present in 8.500 mole of chlorine atoms?
2. Determine the mass (g) of 15.50 mole of oxygen.
3. Determine the number of moles of helium in 1.953 x 108 g of helium.
4. Calculate the number of atoms in 147.82 g of sulfur.
5. Determine the molar mass of Co.
6. Determine the formula mass of Ca3(PO4)2.
IT WOULD BE HELPFUL

Answers

Answer 1

The number of atoms present in 8.500 mole of chlorine atoms can be calculated using Avogadro's number, which is 6.022 x [tex]10^{23}[/tex] atoms per mole. Therefore:

Number of atoms = 8.500 mole x 6.022 x [tex]10^{23}[/tex]atoms/mole

Number of atoms = 5.1177 x [tex]10^{24}[/tex] atoms

Find out the mass (g) of 15.50 mole of oxygen?

The mass of 15.50 mole of oxygen can be calculated using the molar mass of oxygen, which is 16.00 g/mol. Therefore:

Mass = 15.50 mole x 16.00 g/mole

Mass = 248 g

The number of moles of helium in 1.953 x [tex]10^{8}[/tex] g of helium can be calculated using the molar mass of helium, which is 4.00 g/mol. Therefore:

Number of moles = 1.953 x [tex]10^{8}[/tex] g / 4.00 g/mol

Number of moles = 4.883 x [tex]10^{7}[/tex] mol

The number of atoms in 147.82 g of sulfur can be calculated using the molar mass of sulfur, which is 32.06 g/mol, and Avogadro's number. Therefore:

Number of moles = 147.82 g / 32.06 g/mol

Number of moles = 4.608 mol

Number of atoms = 4.608 mol x 6.022 x [tex]10^{23}[/tex] atoms/mol

Number of atoms = 2.773 x [tex]10^{24}[/tex] atoms

The molar mass of Co (cobalt) is 58.93 g/mol.

The formula mass of Ca3(PO4)2 can be calculated by adding the atomic masses of each element in the compound. The atomic masses are:

Ca = 40.08 g/mol

P = 30.97 g/mol

O = 16.00 g/mol

Formula mass = (3 x Ca) + (2 x P) + (8 x O)

Formula mass = (3 x 40.08 g/mol) + (2 x 30.97 g/mol) + (8 x 16.00 g/mol)

Formula mass = 310.18 g/mol

to know more about atoms

brainly.com/question/30898688

#SPJ1


Related Questions

1. Find the square root of each of the following numbers: (i) 152.7696​

Answers

To find the square root of 152.7696, we can use a calculator or apply a method like the following:

1. Start by making pairs of digits from the right: 15, 27, 69, 6.

2. Find the largest integer whose square is less than or equal to the first pair, which is 15. Since 3^2 = 9 < 15 and 4^2 = 16 > 15, the integer we are looking for is 3.

3. Write the digit 3 as the first digit of the square root.

4. Subtract the square of 3 from 15 to get 6.

5. Bring down the next pair of digits, 27, and append them to 6 to get 627.

6. Double the first digit of the current root estimate (which is 3) to get 6.

7. Find the largest digit to fill in the blank in "3_ × _ = 6" such that the resulting product is less than or equal to 627. This digit is 7, since 3×7 = 21 < 627 and 3×8 = 24 > 627.

8. Write down the digit 7 as the second digit of the root estimate.

9. Subtract the square of 37 from 627 to get 60.

10. Bring down the next pair of digits, 69, and append them to 60 to get 6069.

11. Double the first digit of the current root estimate (which is 37) to get 74.

12. Find the largest digit to fill in the blank in "37_ × _ = 606" such that the resulting product is less than or equal to 6069. This digit is 1, since 37×1 = 37 < 6069 and 37×2 = 74 > 6069.

13. Write down the digit 1 as the third digit of the root estimate.

14. Subtract the square of 371 from 6069 to get 152.769.

Therefore, the square root of 152.7696 is approximately 12.36 (rounded to two decimal places).

Helppp on this problem

Answers

The missing angles of the diagram are:

∠1 = 118°

∠2 = 62°

∠3 = 118°

∠4 = 30°

∠5 = 32°

∠6 = 118°

∠7 = 30°

∠8 = 118°

How to find the missing angles?

Supplementary angles are defined as two angles that sum up to 180 degrees. Thus:

∠1 + 62° = 180°

∠1 = 180 - 62

∠1 = 118°

Now, opposite angles are congruent and ∠2 is an opposite angle to 62°. Thus: ∠2 = 62°.

Similarly: ∠3 = 118° because it is congruent to ∠1

Alternate angles are congruent and ∠5 is an alternate angle to 32°. Thus:

∠5 = 32°

Sum of angle 4 and 5 is a corresponding angle to ∠2 . Thus:

∠4 + ∠5 = 62

∠4 + 32 = 62

∠4 = 30°

This is an alternate angle to ∠7 and as such ∠7 = 30°

Sum of angles on a straight line is 180 degrees and as such:

∠8 = 180 - (30 + 32)

∠8 = 118° = ∠6 because they are alternate angles

Read more about Missing Angles at: https://brainly.com/question/28293784

#SPJ1

true or false? the rule of thumb for estimating the time of completion is 1.5 times what you originally think it will be.

Answers

Given statement: The rule of thumb for estimating the time of completion is 1.5 times what you originally think it will be.

Given statement is True.

The reason for this rule of thumb is that tasks often take longer than initially estimated due to unforeseen challenges, dependencies, and other factors that can cause delays.

By estimating the time of completion as 1.5 times the original estimate, you are accounting for these potential delays and allowing for a more realistic timeline.

This rule of thumb is not a hard and fast rule, and there will be cases where tasks take less or more time than 1.5 times the original estimate. However, it can be a helpful guideline for project planning and resource allocation, as it helps to ensure that sufficient time and resources are allocated to complete tasks within a realistic timeframe.

For similar question on estimating.

https://brainly.com/question/597381

#SPJ11

brainlist
show all steps nd i will make u brainlist

Answers

Step-by-step explanation:

Again, using similar triangle ratios

7.2 m  is to 2.4 m  

     as   AB is to  12.0 m

7.2 / 2.4  = AB/12.0    Multiply both sides of the equation by 12

12 *   7.2 / 2.4   = AB = 36.0 meters

Can someone help me asap? It’s due tomorrow. I will give brainiest if it’s correct.

A. 23

B. 61

C. 37

D. 14

Answers

Answer:  the answer is A. 14.

Step-by-step explanation: In each trial of the reenactment, Scott chooses one card from the stack and records its digit. Based on the given data, a digit of or 1 speaks to a objective scored, and a digit of 2 through 9 speaks to a missed endeavor.

Out of the 5 endeavors per amusement, on the off chance that Scott scores precisely 2 objectives, it implies he missed 3 endeavors. Subsequently, the likelihood of this occasion can be calculated as:

P(exactly 2 objectives) = (0.2)²(0.8)³ = 0.008192

This likelihood can be utilized to discover the anticipated number of diversions in which Scott scores precisely 2 objectives, by duplicating it by the overall number of diversions reenacted:

Anticipated number of recreations = P(exactly 2 objectives) × Add up to number of recreations = 0.008192 × 84 ≈ 0.68

Adjusting to the closest entire number, we get that Scott is anticipated to score precisely 2 objectives in 1 diversion out of the 84 recreated diversions.

if x is a matrix of centered data with a column for each field in the data and a row for each sample, how can we use matrix operations to compute the covariance matrix of the variables in the data, up to a scalar multiple?

Answers

To compute the covariance matrix of the variables in the data, the "matrix-operation" which should be used is ([tex]X^{t}[/tex] × X)/n.

The "Covariance" matrix is defined as a symmetric and positive semi-definite, with the entries representing the covariance between pairs of variables in the data.

The "diagonal-entries" represent the variances of individual variables, and the off-diagonal entries represent the covariances between pairs of variables.

Step(1) : Compute the transpose of the centered data matrix X, denoted as [tex]X^{t}[/tex]. The "transpose" of a matrix is found by inter-changing its rows and columns.

Step(2) : Compute the "dot-product" of [tex]X^{t}[/tex] with itself, denoted as [tex]X^{t}[/tex] × X.

The dot product of two matrices is computed by multiplying corresponding entries of the matrices and summing them up.

Step(3) : Divide the result obtained in step(2) by the number of samples in the data, denoted as "n", to get the covariance matrix.

This step scales the sum of the products by 1/n, which is equivalent to taking the average.

So, the covariance matrix "C" of variables in "centered-data" matrix X can be expressed as: C = ([tex]X^{t}[/tex] × X)/n.

Learn more about Covariance Matrix here

https://brainly.com/question/30481066

#SPJ4

The given question is incomplete, the complete question is

Let X be a matrix of centered data with a column for each field in the data and a row for each sample. Then, not including a scalar multiple, how can we use matrix operations to compute the covariance matrix of the variables in the data?

at a booth at the school carnival in past years, they've found that 22% of students win a stuffed toy ($3.60), 16% of students win a jump rope ($1.20), and 6% of students win a t-shirt ($7.90). the remaining students do not win a prize. if 150 students play the game at the booth, how much money should the carnival committee expect to pay for prizes for that booth?: *

Answers

For a percentage data of students who play the different game and win the prize, the expected amount to pay for prizes for that booth by the carnival committee is equals to the $218.70.

We have a booth of school carnival in past years, The percentage of students win a stuffed toy = 22%

The percentage of students win a jump rope = 16%

The percentage of students win a t-shirt

= 6%

The winning amount for stuffed toy game = $ 3.60

The winning amount for jump rope game = $1.20

The winning amount for t-shirt game

= $7.90

The remaining students do not win a prize. Now, total number of students play the game at the booth = 150

So, number of students who win stuffed toy = 22% of 150 = 33

Number of students who win jump rope = 16% of 150 = 24

Number of students who win stuffed toy

= 6% of 150 = 9

For determining the expected pay using the simple multiplication formula. Total expected pay for prizes for that booth is equals to the sum of resultant of multiplcation of number of students who play a particular game into pay amount for that game. That is total excepted pay in dollars = 3.60 × 33 + 1.20 × 24 +7.90 × 6

= 218.7

Hence required value is $218.70.

For more information about percentage, visit:

https://brainly.com/question/843074

#SPJ4

for a certain type of hay fever, medicine h has a 30% probability of working. in which distributions does the variable x have a binomial distribution? select each correct answer.

Answers

The distribution in which variable x has binomial distribution are as follow,

Option A) When the medicine is tried with two patients, X is the number of patients for whom the medicine worked.

Option D) When the medicine is tried with six patients, X is the number of patients for whom the medicine worked.

When the medicine is tried with two patients, X is the number of patients for whom the medicine worked.

This variable X follows a binomial distribution .

Because there are two independent trials two patients.

With a constant probability of success 30%.

And the outcome of one trial doesn't affect the outcome of the other.

When the medicine is tried with six patients, X is the number of patients for whom the medicine does not work.

This variable X does not follow a binomial distribution.

Because the probability of success is not constant it's the complement of 30%, which is 70%.

Also, the outcome of one trial affects the outcome of the other trials.

As there are only six patients .

Number of patients for whom medicine does not work depends on number of patients for whom it worked.

When the medicine is tried with six patients, X is the number of patients for whom the medicine worked.

This variable X follows a binomial distribution.

Because there are six independent trials six patients with a constant probability of success (30%) .

The outcome of one trial doesn't affect the outcome of the other.

When the medicine is tried with two patients, X is the number of doses each patient needs to take.

This variable X does not follow a binomial distribution.

Because it's not a count of successes out of a fixed number of independent trials.

But rather a continuous variable that can take any non-negative value.

Learn more about binomial distribution here

brainly.com/question/12702509

#SPJ4

The above question is incomplete, the complete question is:

For a certain type of hay fever, Medicine H has a 30% probability of working.

In which distributions does the variable X have a binomial distribution?

Select EACH correct answer.

A. When the medicine is tried with two patients, X is the number of patients for whom the medicine worked.

B. When the medicine is tried with six patients, X is the number of patients for whom the medicine does not work.

C. When the medicine is tried with six patients, X is the number of patients for whom the medicine worked.

D. When the medicine is tried with two patients, X is the number of doses each patient needs to take.

Help please, I'm so lost

Answers

I gotchu <3

Since the vertex is (0, -3), the quadratic function can be written in vertex form as:

f(x) = a(x - 0)^2 - 3

Where 'a' is a constant that determines the shape of the parabola. Since the end behavior of the function is y --> - Infinite as x --> - infinite and y --> - Infinite as x --> + infinite, the leading coefficient 'a' must be negative.

So, f(x) = -a(x^2 - 0x) - 3

Now, using the given point (1, -7) on the parabola, we can substitute the coordinates into the function and solve for 'a'.

-7 = -a(1^2 - 0(1)) - 3

-7 = -a - 3

a = 10

Therefore, the quadratic function that satisfies the given characteristics is:

f(x) = -10x^2 - 3

Hope this helps :)

the percentage of the original area of wetlands currently left in the united states is approximately: question 44 options: 10%. 25%. 50%. 65%. 75%.

Answers

The percentage of the original area of wetlands currently left in the United States is approximately 50%. So, the correct

option is 50% (option 3).

Percentages are a way of expressing a proportion or a fraction as a part of 100. It is denoted by the symbol "%".

According to the United States Environmental Protection Agency (EPA), it is estimated that about 50% of the original

wetlands in the contiguous United States have been lost since the 1600s due to human activities such as agriculture,

development, and urbanization. Therefore, the correct answer to the question is 50%.

for such more questions on percentages

https://brainly.com/question/24877689

#SPJ11

The percentage of the original area of wetlands currently left in the United States is approximately 50%.

Wetlands are regions of land where the soil is continually or intermittently soaked with water. Wetlands have a particular hydrology, soil, and vegetation mix that results in specialised ecosystems that offer a variety of ecological functions.

There are many different types of habitats where wetlands can be found, such as coastal locations, interior regions, and high-altitude mountain regions. They come in a variety of shapes, such as marshes, swamps, bogs, fens, and estuaries, and can be freshwater, brackish, or saline.

Wetlands are significant for several reasons. For species, such as migrating birds, amphibians, and fish, they offer crucial habitats. They also aid in removing contaminants from water, lessen the effects of flooding, and give people access to recreational activities. Wetlands are crucial for carbon sequestration as well.
Based on the information provided, the question is asking for the approximate percentage of the original area of wetlands currently left in the United States. The answer is approximately 50%.

Learn more about wetlands here:

https://brainly.com/question/30010590

#SPJ11

Find all of the cube roots of 216i and write the answers in rectangular (standard) form.

Answers

The cube roots of 216 written in the rectangular (standard) form are 3 + 3√3, -3+3√3, and 6.

What is a cube root?

In mathematics, the cube root formula is used to represent any number as its cube root, for example, any number x will have the cube root 3x = x1/3. For instance, 5 is the cube root of 125 as 5 5 5 equals 125.

3√216 = 3√(2x2x2)x(3x3x3)

= 2 x 3 = 6

the prime factors are represented as cubes by grouping them into pairs of three. As a result, the necessary number, which is 216's cube root, is 6.

Therefore, the cube roots of 216 are 3 + 3√3, -3+3√3, and 6.

Learn more about cube root, here:

https://brainly.com/question/31153573

#SPJ1

Which ratio is proportional to 80:60?

16:15

16:12

18:15

18:12

Pls answer quickly

Answers

Answer:

To determine which ratio is proportional to 80:60, we need to simplify the ratio by dividing both terms by their greatest common factor. In this case, the greatest common factor of 80 and 60 is 20, so:

80/20 = 4

60/20 = 3

Therefore, the simplified ratio is 4:3.

Now we can compare this ratio to the given options to see which one is proportional to 4:3:

16:15 is not proportional

16:12 is proportional (since 16/4 = 4 and 12/3 = 4)

18:15 is not proportional

18:12 is proportional (since 18/3 = 6 and 12/2 = 6)

Therefore, the ratio that is proportional to 80:60 is 16:12.

a work system has five continuous stations that have process times of 5, 8, 4, 7, and 8 min/unit respectively. what is the process time of the system?

Answers

The process time of the system is 32 min/unit.

Two bank accounts open with deposits of $1,810 and annual interest rates of 2.5%. Bank A uses simple interest and Bank B uses interest compounded monthly How much more in interest does the account at Bank B earn in 5 years?​

Answers

[tex]~~~~~~ \stackrel{ \textit{\LARGE Bank A} }{\textit{Simple Interest Earned}} \\\\ I = Prt\qquad \begin{cases} I=\textit{interest earned}\\ P=\textit{original amount deposited}\dotfill & \$1810\\ r=rate\to 2.5\%\to \frac{2.5}{100}\dotfill &0.025\\ t=years\dotfill &5 \end{cases} \\\\\\ I = (1810)(0.025)(5) \implies I = 226.25 \\\\[-0.35em] ~\dotfill[/tex]

[tex]~~~~~~ \stackrel{ \textit{\LARGE Bank B} }{\textit{Compound Interest Earned Amount}} \\\\ A=P\left(1+\frac{r}{n}\right)^{nt} \quad \begin{cases} A=\textit{accumulated amount}\\ P=\textit{original amount deposited}\dotfill &\$1810\\ r=rate\to 2.5\%\to \frac{2.5}{100}\dotfill &0.025\\ n= \begin{array}{llll} \textit{times it compounds per year}\\ \textit{monthly, thus twelve} \end{array}\dotfill &12\\ t=years\dotfill &5 \end{cases}[/tex]

[tex]A = 1810\left(1+\frac{0.025}{12}\right)^{12\cdot 5} \implies A \approx 2050.73~\hfill \underset{ interest }{\stackrel{2050.73~~ - ~~1810 }{\approx 240.73}} \\\\[-0.35em] ~\dotfill\\\\ 240.73~~ - ~~226.25 ~~ \approx ~~ \text{\LARGE 14.48}[/tex]

There are only Ured counters and g green counters in a bag. A counter is taken at random from the bag. The probability that the counter is green is 3
7
The counter is put back in the bag. 2 more red counters and 3 more green counters are put in the bag. A counter is taken at random from the bag. The probability that the counter is green is 6
13
Find the number of red counters and the number of green counters that were in the bag originally

Answers

Number of red counters in the original bag is 3, and the number of green counters is 7.

Let's use algebra to solve the problem. Let U be the number of red counters and G be the number of green counters originally in the bag.

From the first part of the problem, we know that

Probability (selecting a green counter) = G / (U + G) = 3/7

Solving for U in terms of G, we get

U = (7G - 3G) / 3 = 4G/3

So we know that there were 4G/3 red counters and G green counters in the bag originally. But since the number of counters must be a whole number, we can assume that there were 4R red counters and 3G green counters originally, where R and G are both integers and R + G is the total number of counters.

After adding 2 red and 3 green counters, the number of counters in the bag is now R + 2 + G + 3 = R + G + 5.

From the second part of the problem, we know that

P(selecting a green counter) = (G + 3) / (R + G + 5) = 6/13

Solving for R in terms of G, we get

R = (13G - 9G - 15) / 7 = 4G/7 - 15/7

Since R must be an integer, we can try different values of G to see if R is an integer. For example, if G = 7, then R = 3 and the total number of counters is 10.

Learn more about probability here

brainly.com/question/11234923

#SPJ4

The given question is incomplete, the complete question is:

There are only U red counters and G green counters in a bag. A counter is taken at random from the bag. The probability that the counter is green is 3/7. The counter is put back in the bag. 2 more red counters and 3 more green counters are put in the bag. A counter is taken at random from the bag. The probability that the counter is green is 6/13. Find the number of red counters and the number of green counters that were in the bag originally

hat is the maximum speed of a point on the outside of the wheel, 15 cm from the axle?

Answers

It depends on the rotational speed of the wheel. To calculate this speed, we need to know the angular velocity of the wheel.

The maximum speed of a point on the outside of the wheel, 15 cm from the axle, if we assume that the wheel is rotating at a constant rate, we can use the formula v = rω, where v is the speed of the point on the outside of the wheel, r is the radius of the wheel (15 cm in this case), and ω is the angular velocity of the wheel. Therefore, the maximum speed of a point on the outside of the wheel would be directly proportional to the angular velocity of the wheel.
The formula to calculate the maximum linear speed (v) is:
v = ω × r
where v is the linear speed, ω is the angular velocity in radians per second, and r is the distance from the axle (15 cm, or 0.15 meters in this case).
Once you have the angular velocity (ω) of the wheel, you can plug it into the formula and find the maximum speed of a point on the outside of the wheel.

Learn more about rotational speed here: brainly.com/question/17025846

#SPJ11

Homework 18.1.-trigonometric ratios

Find the 3 trigonometric ratios. If needed, reduce fractions.

Answers

Step-by-step explanation:

rotate the triangle in your mind (or as actual picture on your phone or computer), so that the right angle is the bottom right or bottom left, and C being the opposite bottom angle.

then we see

28 = cos(C) × 35

21 = sin(C) × 35

and so,

sin(C) = 21/35 = 3/5

cos(C) = 28/35 = 4/5

tan(C) = sin(C)/cos(C) = 3/5 / 4/5 = 3/4

compute the residuals. (round your answers to two decimal places.) xi yi residuals 6 6 11 7 15 12 18 20 20 30 (c) develop a plot of the residuals against the independent variable x. do the assumptions about the error terms seem to be satisfied?

Answers

The estimated regression equation for the given data is y = -30.7 + 3.409x

To develop an estimated regression equation for the given data, we need to use the method of least squares.

The formula for the slope of the regression line is given by:

b = ∑(xi - x)(yi - y) / ∑(xi - x)²

where xi and yi are the individual values of the two variables, x and y are their respective means.

The formula for the intercept of the regression line is given by:

a = y - b × x

where a is the intercept and b is the slope.

Using the given data, we can calculate the values of x, y, b, and a as follows

x = (6 + 11 + 15 + 18 + 20) / 5 = 14

y = (7 + 9 + 12 + 21 + 30) / 5 = 15.8

∑(xi - x)(yi - y) = (6 - 14)(7 - 15.8) + (11 - 14)(9 - 15.8) + (15 - 14)(12 - 15.8) + (18 - 14)(21 - 15.8) + (20 - 14)(30 - 15.8) = 306.8

∑(xi - x)² = (6 - 14)² + (11 - 14)² + (15 - 14)² + (18 - 14)² + (20 - 14)² = 90

b = ∑(xi - x)(yi - y) / ∑(xi - x)² = 306.8 / 90 = 3.409

a = y - b × x = 15.8 - 3.409 × 14 = -30.7

Learn more about regression equation here

brainly.com/question/14184702

#SPJ4

The given question is incomplete, the complete question is:

Given are data for two variables, x and y. Develop an estimated regression equation for these data.

Copy
State
This
prior
any r
perm
Man
avv
ttr
opy
om
and
av
Marven and three friends are renting a car for a trip. Rental prices are
shown in the table.
Item
PART B
Small car rental fee
-seats 4 passengers
Full-size car rental fee
-seats 4 passengers
Insurance
Price
465=25x
$39/day
$49/day
$21/day
25
(X=18.6
-198
018.6
1465
LIS
If they still use the coupon, how many days could they rent the small car
with insurance if they have $465 to spend?

Answers

Since they can't rent for a fraction of a day, the maximum number of days they can rent the small car with insurance is 10 days.

Insurance calculation.

The total cost of renting a small car with insurance is:

$465 = $25x + $21x

Simplifying and solving for x, we get:

$465 = $46x

x = 10.11

Since they can't rent for a fraction of a day, the maximum number of days they can rent the small car with insurance is 10 days.

Learn more about insurance below.

https://brainly.com/question/25221455

#SPJ1

what is the 4th term/number of (a+b)^9, pascal’s triangle?

Answers

Step-by-step explanation:

hope this will help you Thanks

A company is designing a new cylindrical water bottle. The volume of the bottle will be 150 cm^3. The height of the water bottle is 8.9 cm. What is the radius of the water​ bottle? Use 3.14 for π.

Answers

The radius of the water bottle which is in cylindrical shape is approximately 2.12 cm.

What is the cylindrical shape?

A cylinder is a three-dimensional shape that consists of two congruent, parallel circular bases that are connected by a curved lateral surface. The lateral surface of the cylinder is formed by a rectangle that is wrapped around the circular bases.

A cylinder can be thought of as a circular prism, where the bases are circles and the lateral surface is curved. The height of a cylinder is the perpendicular distance between the two bases, and the radius is the distance from the center of the base to the edge of the circular base.

According to the given information

The formula for the volume of a cylinder is V = πr^2h, where V is the volume, r is the radius, and h is the height. We can rearrange this formula to solve for the radius:

r = √(V/πh)

Substituting the given values, we have:

r = √(150/π(8.9))

r ≈ 2.12 cm (rounded to two decimal places)

Therefore, the radius of the water bottle is approximately 2.12 cm.

To know more about the Cylinder visit:

brainly.com/question/15891031

#SPJ1

John buys new baseball equipment for $2000. The purchase made is with a credit card that has a 19% APR. John makes a $150 payment monthly. How many months will it take John to pay off the balance?

Answers

It will take John approximately 17 months to pay off the balance.

What is simple interest?

Simple Interest (S.I.) is the method of calculating the interest amount for a particular principal amount of money at some rate of interest.

Assuming that John does not use the credit card for any other purchases and that the credit card company uses a simple interest calculation method, we can use the following steps to calculate the number of months it will take John to pay off the balance:

Calculate the monthly interest rate by dividing the annual percentage rate (APR) by 12:

Monthly interest rate = 19% / 12 = 0.01583

Calculate the monthly finance charge by multiplying the outstanding balance by the monthly interest rate:

Monthly finance charge = $2000 x 0.01583 = $31.66

Subtract the monthly payment from the monthly finance charge to get the amount that will be applied to the outstanding balance:

Payment applied to balance = $150 - $31.66 = $118.34

Divide the outstanding balance by the payment applied to balance to get the number of months it will take to pay off the balance:

Number of months to pay off balance = $2000 / $118.34 = 16.9

(rounded up to the nearest whole number)

Therefore, it will take John approximately 17 months to pay off the balance.

To learn more about the simple interest visit:

brainly.com/question/20690803

#SPJ1

Identify the expression that is not equivalent to 6x + 3.

Answers

The resultant value of the given expression x² + 10x + 24 when x = 3 is (C) 63.

What are expressions?

A finite collection of symbols that are properly created in line with context-dependent criteria is referred to as an expression, sometimes known as a mathematical expression.

Expressions in writing are made using mathematical operators such as addition, subtraction, multiplication, and division.

For instance, "4 added to 2" will have the mathematical formula 2+4.

So, we have the expression:

= x² + 10x + 24

Now, solve when x = 3 as follows:

= x² + 10x + 24

= 3² + 10(3) + 24

= 9 + 30 + 24

= 63

Therefore, the resultant value of the given expression x² + 10x + 24 when x = 3 is (C) 63.

Know more about expressions here:

https://brainly.com/question/723406

#SPJ1

Correct question:

Evaluate the expression when x = 3.

x² + 10x + 24

a. 81

b. 86

c. 63

d. 60

can someone give me the answers to these 5?? pleaseee!!

Answers

The MAD of the hourly wages given would be $ 0.48. The range would be $ 2.00. Q1 would be $8.25. Q3 would then be $9.25. The IQR would be $1.00

How to find the number summaries ?

Calculate the MAD:

First, find the mean of the data set:

mean = (sum of all values) / (number of values)

mean = (8.25 + 8.50 + 9.25 + 8.00 + 10.00 + 8.75 + 8.25 + 9.50 + 8.50 + 9.00) / 10

mean = 88.00 / 10 = 8.80

Then, find the mean of these absolute deviations:

MAD = (sum of absolute deviations) / (number of values)

MAD = (0.55 + 0.30 + 0.45 + 0.80 + 1.20 + 0.05 + 0.55 + 0.70 + 0.30 + 0.20) / 10

MAD = 4.10 / 10 = 0.41

Calculate the range:

range = maximum value - minimum value

range = 10.00 - 8.00 = 2.00

Find Q1 and Q3:

{8.00, 8.25, 8.25, 8.50, 8.50, 8.75, 9.00, 9.25, 9.50, 10.00}

Q1 is the median of the lower half, and Q3 is the median of the upper half.

Lower half: {8.00, 8.25, 8.25, 8.50, 8.50}

Upper half: {8.75, 9.00, 9.25, 9.50, 10.00}

Q1 = median of lower half = 8.25

Q3 = median of upper half = 9.25

Calculate the IQR:

IQR = Q3 - Q1

IQR = 9.25 - 8.25 = 1.00

Find out more on MAD at https://brainly.com/question/3250070

#SPJ1

A frog catches insects for their lunch. The frog likes to eat flies and mosquitoes in a certain ratio, which the diagram shows.
A tape diagram with 2 tapes of unequal lengths. The first tape has 3 equal parts. A curved bracket above the first tape is labeled Flies. The second tape has 7 equal parts of the same size as in the first tape. A curved bracket below the second tape is labeled Mosquitoes.
A tape diagram with 2 tapes of unequal lengths. The first tape has 3 equal parts. A curved bracket above the first tape is labeled Flies. The second tape has 7 equal parts of the same size as in the first tape. A curved bracket below the second tape is labeled Mosquitoes.
The table shows the number of flies and the number of mosquitoes that the frog eats for two lunches.
Based on the ratio, complete the missing values in the table.
Day Flies Mosquitoes
Monday
15
1515
Tuesday
14
1414

Answers

To find the missing values in the table, we need to use the ratio of flies to mosquitoes, which is 3:7. This means that for every 3 flies, the frog eats 7 mosquitoes.

On Monday, the frog ate 15 insects in total. Let x be the number of flies the frog ate. Then the number of mosquitoes the frog ate would be 15 - x. We can set up the equation:

x/3 = (15 - x)/7

Solving for x, we get:

x = 45/10 = 4.5

Since the number of flies and mosquitoes must be whole numbers, we can round up the number of flies to 5 and calculate the number of mosquitoes as 15 - 5 = 10. So, the missing values in the table are:

Day | Flies | Mosquitoes
--------------------------
Monday | 5 | 10
Tuesday| 4 | 14

in a standard additions method workup what information from the linear regression is most closely related to the unknown concentration? (used to determine it)

Answers

In a standard additions method workup the information from the linear regression that is most closely related to the unknown concentration is this:  the intercept of the linear regression line.

What information is closest to the unknown concentration?

In the standard additions method workup, the information that is most closely related to the unknown concentration is the intercept of the linear regression line.

The reason why this is the case is that the intercept represents the y-value of the regression line where the line crosses the y-axis. This y-axis is the value of the dependent variable when the independent variable or concentration is zero. So, by solving for the intercept, we can determine the concentration of the unknown sample.

Learn more about linear regression here:

https://brainly.com/question/25987747

#SPJ1

which of the following is a correct statement regarding the null hypothesis? the null hypothesis is sometimes called the alternative hypothesis. the null hypothesis is the one the researcher cares the most about. the null hypothesis claims the opposite of what the researcher believes. the null hypothesis is usually more accurate than the research hypothesis.

Answers

The correct statement regarding the null hypothesis is: the null hypothesis claims the opposite of what the researcher believes.

The null hypothesis is the claim that no relationship exists between two sets of data or variables being analyzed. The

null hypothesis is that any experimentally observed difference is due to chance alone, and an underlying causative

relationship does not exist, hence the term "null".

In research, the null hypothesis is a statement of no effect or no relationship between variables, while the alternative

hypothesis represents the effect or relationship the researcher is interested in demonstrating.

The purpose of statistical testing is to determine whether there is enough evidence to reject the null hypothesis in

favor of the alternative hypothesis.

for such more question on null hypothesis

https://brainly.com/question/25263462

#SPJ11

you are thinking of combining designer whey and muscle milk to obtain a 7-day supply that provides exactly 262 grams of protein and 54 grams of carbohydrates. how many servings of each supplement should you combine in order to meet your requirements?

Answers

We need approximately 12 servings of Designer Whey and 2 servings of Muscle Milk to obtain a 7-day supply that provides exactly 262 grams of protein and 54 grams of carbohydrates.

Let x be the number of servings of Designer Whey and y be the number of servings of Muscle Milk needed to obtain a 7-day supply that provides exactly 262 grams of protein and 54 grams of carbohydrates.

From the information given, we know that each serving of Designer Whey provides 20 grams of protein and 3 grams of carbohydrates, and each serving of Muscle Milk provides 16 grams of protein and 9 grams of carbohydrates.

Therefore, we can set up the following system of equations:

20x + 16y = 262

3x + 9y = 54

To solve for x and y, we can use any method of solving a system of equations. For example, we can use substitution:

From the second equation, we can solve for x in terms of y:

x = (54 - 9y)/3 = 18 - 3y

Substituting this into the first equation, we get:

20(18 - 3y) + 16y = 262

Simplifying, we get:

80y = 182

Solving for y, we get:

y = 2.275

Substituting this into the equation x = 18 - 3y, we get:

x = 12.175

To learn more about equations click on,

https://brainly.com/question/29118974

#SPJ4

An investment of $600 is made into an account that earns 6. 5% annual simple interest for 15

years. Assuming no other deposits or withdrawals are made, what will be the balance in the

account?

Answers

According to the investment, after 15 years, the balance in the account would be $1185.

To calculate the final balance after 15 years, we can use the formula for simple interest:

Simple Interest = Principal x Interest Rate x Time

In this case, the principal is $600, the interest rate is 6.5%, and the time is 15 years.

Simple Interest = $600 x 0.065 x 15

Simple Interest = $585

So the investment of $600 earns $585 in simple interest over 15 years. To find the final balance, we add the interest earned to the initial investment:

Final Balance = Principal + Simple Interest

Final Balance = $600 + $585

Final Balance = $1185

To know more about investment here

https://brainly.com/question/365124

#SPJ4

WILL MARK AS BRAINLEIST!!
Question in picture!!

Note: The graph above represents both functions “f” and “g” but is intentionally left unlabeled

Answers

Answer:

f(x) is the blue graph, g(x) is the red graph.

x^2 - 3x + 17 - (2x^2 - 3x + 1) = 16 - x^2

16 - x^2 = 0 when x = -4, 4

So the area between these two graphs is (using the TI-83 graphing calculator):

fnInt (16 - x^2, x, -4, 4) = 85 1/3

Other Questions
The function V(t) = 30000(0.85) value V(t) represents the values v(t) of Nancy's car after t years. What is the depreciation rate of Nancy's car? at a certain temperature the solubility of lead(ii) iodide is 0.064 g/100 ml. what is the solubility product of lead(ii) iodide at this temperature? provide your answer rounded to 2 significant figures. What is the current balance between the government protecting the public from terrorist threats while also protecting peoples individual rights burke's corner currently sells blue jeans and t-shirts. management is considering adding fleece tops to its inventory to provide a cooler weather option. the tops would sell for $46 each with expected sales of 4,650 tops annually. by adding the fleece tops, management feels the firm will sell an additional 320 pairs of jeans at $58 a pair and 455 fewer t-shirts at $19 each. the variable cost per unit is $29 on the jeans, $9 on the t-shirts, and $24 on the fleece tops. with the new item, the depreciation expense is $26,000 a year and the fixed costs are $79,500 annually. the tax rate is 24 percent. what is the project's operating cash flow? which of the following are relative measures of sales and profits? (choose every correct answer.) multiple select question. a firm's net profit from lowered prices a firm's growth as compared to other companies a firm's total global sales a firm's increase in sales over the prior year Do a comparison and contrast on John Donne poem talking about his mistress going to bed and Marvel Andrews 'To coy his mistress' the major flaw of the linear probability model is that a. the actuals can only be 0 and 1, but the predicted are almost always different from that. b. the regression r2 cannot be used as a measure of fit. c. people do not always make clear-cut decisions. d. the predicted values can lie above 1 and below 0. (Cost of preferred stock) The preferred stock of Texas Southern Power Company sells for $41 and pays $7 in dividends. The net price of the security after issuance costs is $36.08 . What is the cost of capital for the preferred stock? Find the measures of angle a and B. Round to the nearest degree. in recent years, adopted children (and their adoptive families) are more likely to communicate with the childrens birth family, an arrangement known as a(n) ___________ . Write an equation for the polynomial graphed below managers can reduce the need for organizational rules and regulations by hiring the right people, providing training, developing management role models, and creating blank systems. multiple choice question. reward rule-based control corporate governance collins manufacturing has the following information: common stock is 2.5 million shares with a current price of $42 per share; the beta of the stock is 1.5; the standard deviation of the stock is 10.5%. market: the us treasury bill is yielding 2.8% and the expected return on the market is 10.8%. the corporate tax rate is 38%. what is the firm's expected return on equity? in operant conditioning, the subject's behavior determines an outcome and is affected by the outcome. a. true b. false a teacher who is culturally curious and responsive recognizes that all people are influenced by their ________, and that variations within cultures are as significant as variations across cultures.] 50 points +brainlist (there's going to be 3 more added on my profile with the same points(which type of process is this?chemicalphysicalnuclear In Circle C, the diameter AB bisects the chord EF.If m The monthly cost (in dollars) of a long-distance phone plan is a linear function of the total calling time (in minutes). The monthly cost for 35 minutes of calls is $16.83 and the monthly cost for 52 minutes is $18.87. What is the monthly cost for 39 minutes of calls? Dantzler Corporation is a fast-growing supplier of office products. Analysts project the following free cash flows (FCFs) during the next 3 years, after which FCF is expected to grow at a constant 5% rate. Dantzler's WACC is 16%.a) Year 0 FCF N/Ab) Year 1 FCF -$15 millionc) Year 2 FCF $28 milliond) Year 3 FCF $46 milliona. What is Dantzler's horizon, or continuing value? (Hint: Find the value of all free cash flows beyond Year 3 discounted back to Year 3.) Round your answer to 2 decimal places. Enter your answers in millions.b. What is the firm's value today? Round your answer to 2 decimal places. Enter your answers in millions. Do not round intermediate calculations.c. Suppose Dantzler has $141 million of debt and 7 million shares of stock outstanding. What is your estimate of the current price per share? Round your answer to 2 decimal places. the nurse is caring for a client with diabetes who has an infection. the nurse creates a plan of care for the client based on a knowledge of the hypothalamic-pituitary response to stress by including which nursing intervention in the plan of care?